سوال المپیادی(سوال و حل آن ها با همکاری هم)

وضعیت
موضوع بسته شده است.
پاسخ : سوال المپیادی(سوال و حل آن ها با همکاری هم)

نه اصلا هيچ چيز معلوم نيست !ما فقط ميدونيم يه سرى جعبه هستن كه توشون سيب و پرتقاله!البته محتويات جعبه هارو ميدونيم ها!
 
پاسخ : سوال المپیادی(سوال و حل آن ها با همکاری هم)

اين سوال كه مى گم قرار بوده سوال هندسه ى مرحله ٢ رياضى ٢ سال پيش باشه!m روى كمان bc روى دايره ى محيطى مثلث abc است!از m به ab,ac
عمود مى كنيم و پاى عمود ها را h,k
مى ناميم!m كجاى bc باشد كه mh +mk
حداكثر شود؟
 
پاسخ : سوال المپیادی(سوال و حل آن ها با همکاری هم)

سلام به همه من یه راه رفتم تقریبا کامله اما نه کاملا!!!فرض کنید عمود وارد بر ABان را در Xقطع کند و وای هم به همین گونه!!فرض کنید XY
BC را در f قطع کند ثابت کنید BFMXو FMCY محاطیست نتیجه بگیرید مثلث های BXMوMYCمتشابه اند!! ;;)(اگر بعضی را نفهمیدید چرا فکر کنید اگر باز نفهمیدید بگید تا بگم چرا!!)و بدست می اید:
MX+MY=(MB+Mc)*MB/MC
حالا چی کار کنیم؟؟ :-?
 
پاسخ : سوال المپیادی(سوال و حل آن ها با همکاری هم)

امروز یه چیز باحالتر کشف کردم ببینید mx+myبرابر است با :mb+mc*(am/2r)=mx+my
به درد میخوره؟؟
 
پاسخ : سوال المپیادی(سوال و حل آن ها با همکاری هم)

نه!اين هايي كه يافتى به درد حل سوال نمى خوره!سوال يكم سخته!اگه مى خوايد راهنمايي مى كنم
 
پاسخ : سوال المپیادی(سوال و حل آن ها با همکاری هم)

سلام بگو عیب نداره!!
 
پاسخ : سوال المپیادی(سوال و حل آن ها با همکاری هم)

مكان هندسى نقاطى رو توى يه زاويه پيدا كن كه مجموع طول عمود هاى وارد بر اضلاع زاويه از اون نقاط يه مقدار ثابت باشه!
 
پاسخ : سوال المپیادی(سوال و حل آن ها با همکاری هم)

به نقل از killer :
salam age mishe chand ta soale jabre ghashang bezarid k hame estefade konim

کهکشان راه دوغی بیش از 6^10 ستاره دارد ، ثابت کنید در هر لحظه فاصله ی 2 به 2 ی نقاط حداقل 79 عدد متمایز است.؟
 
پاسخ : سوال المپیادی(سوال و حل آن ها با همکاری هم)

در آنجا كه ما بوديم اسم اين تركيبيات بود!يك سوال جبر هم دارم!چند جمله اى p با ضرايب صحيح داده شده است،به ازاى عدد هاى صحيح حداكثر چند عدد صحيح متوالى مى تواند در برد اين چند جمله اى باشد!
 
پاسخ : سوال المپیادی(سوال و حل آن ها با همکاری هم)

به نقل از حامد مهدوی :
در آنجا كه ما بوديم اسم اين تركيبيات بود!يك سوال جبر هم دارم!چند جمله اى p با ضرايب صحيح داده شده است،به ازاى عدد هاى صحيح حداكثر چند عدد صحيح متوالى مى تواند در برد اين چند جمله اى باشد!

این سوال از طریق ترکیبیات حل نمیشییی

یعنی‌ در وقی راه حل خاصی‌ نداره!!!!!
 
پاسخ : سوال المپیادی(سوال و حل آن ها با همکاری هم)

اين دقيقا سوال لانه كبوتريه!فينگيليش هم ننويسيد!خلاف قوانينه!
 
پاسخ : سوال المپیادی(سوال و حل آن ها با همکاری هم)

با برهان خلف اثبات میشه و اصل لانه کبوتری...
 
پاسخ : سوال المپیادی(سوال و حل آن ها با همکاری هم)

به نقل از samin159 :
با برهان خلف اثبات میشه و اصل لانه کبوتری...
همه میدونیم چه جوری اثبات میشه !
از کی تاحالا اصل لانه کبوتری شده جبر؟
 
پاسخ : سوال المپیادی(سوال و حل آن ها با همکاری هم)

من موقعی که این سوالو گذاشتم به کامنت قبلی توجه نکردم که توش جبر بود! به بزرگواریه خودتون ببخشید!!!!
 
پاسخ : سوال المپیادی(سوال و حل آن ها با همکاری هم)

واسه سوال هندسه : (max ( MH + MK

اگه شعاع دایره محیطیه مثلثو ۱/۲ بگیریم و زاویه MAB را x بگیریم

داریم: ( AM = sin ( x+ C

پس ( HM = sin x * AM = sin x * sin (x + C

با توجه به رابطهٔ :

sin x * sin y * ۲ = cos x-y - cos x+y

داریم HM=

۱/۲ *(( cos C - cos (۲x+ C )=

به طریق مشابه

این بر قرار میدهیم

( AM = sin ( B + A-x

MK = sin A-x * AM = sin A-x * sin B + A-x

۱/۲* (( cos (B) - cos ( ۲A+B-۲x)=

در عبارت‌های بالا cos C و cos B که همواره ثابت هستند

پس باید کاری کنیم که cos ۲A+ B-۲x + cos ۲x - C مینیمم شود

از رابطهٔ cos x * cos y * ۲ = cos x+y + cos x-y

عبارت فوق میشود
cos (A+B/2+C/2) * cos (A+B/2-C/2-2x) * 2
چون A<180 پس A+B/2+C/2<180
و بدیهیست که A+B/2+C/2>90
پس (cos(A+B/2+C/2 همواره منفیست
پس باید (cos(A+B/2-C/2-2x برابر ۱ باشد تا مقدار خواسته شده مینیمم شود یعنی x=A/2+B/4-C/4
بدیهی هم هست که M بین B C میفته
پس مکان M پیدا شد
این حل تقدیم به عرفان که دفعه بعد من اومدم بهشون هندسه درس بدم گوش کنه !اینجوری آبروریزی نکنه
 
پاسخ : سوال المپیادی(سوال و حل آن ها با همکاری هم)

خیلی تمیز تر از این حرف ها حل میشه!زاویه ی A رو در نظر بگیرید ،مکان هندسی نقاط درون زاویه ی A که مجموع طول عمود های مرسوم به اضلاع زاویه از اون نقطه ها مقدار ثابتیه برابر هستش با پاره خطی که قاعده ی مثلث متساوی الساقینی هستش که A زاویه ی رو به روی قاعدشه!حالا هر چی این خط رو بیاریم پایین تر به مجموع طول عمود ها بیشتر میشه!واضحه که ماکسیموم جایی رخ میده که این مکان هندسی به دایره مماس شه!
 
پاسخ : سوال المپیادی(سوال و حل آن ها با همکاری هم)

مهم اینه که حل شده!
در ضمن من اغلب هندسه ها را با مثلثات حل میکنم
 
پاسخ : سوال المپیادی(سوال و حل آن ها با همکاری هم)

چند دنباله به طول n از اعداد 0و1و2و3 وجود دارد با این شرط که هرکدام شامل تعداد زوجی 0 و تعداد فردی 1 باشد؟ (روی تعداد 2ها و 3ها شرطی نداریم).


منبع: مسائل برگزیده ریاضی از سراسر دنیا ترجمه ی امیر آجرلو
 
پاسخ : سوال المپیادی(سوال و حل آن ها با همکاری هم)

به نقل از y.a :
چند دنباله به طول n از اعداد 0و1و2و3 وجود دارد با این شرط که هرکدام شامل تعداد زوجی 0 و تعداد فردی 1 باشد؟ (روی تعداد 2ها و 3ها شرطی نداریم).


منبع: مسائل برگزیده ریاضی از سراسر دنیا ترجمه ی امیر آجرلو
فکر کنم با بازگشتی بشه حلش کرد
 
پاسخ : سوال المپیادی(سوال و حل آن ها با همکاری هم)

سلام یه سوال داشتم
x و y اعداد حقیقی اند بصورتی که x^2 + y^2 = 1
max(3x+4y) چنده
 
وضعیت
موضوع بسته شده است.
Back
بالا